Chapter 23: Pharmacotherapy of Muscle Spasms and Spasticity, Musculoskeletal

Réussis tes devoirs et examens dès maintenant avec Quizwiz!

...

8. a. Ossification is the stage of fracture healing when there is clinical union and enough strength to prevent movement at the fracture site. Remodeling is the return to preinjury structural strength and shape. Consolidation is when the distance between bone fragments eventually closes and radiologic union first occurs. The callus formation stage appears by the end of the second week of injury, when minerals and new bone matrix are deposited in the osteoid produced in the granulation tissue stage.

...

9. a, b, c, e. When the remodeling stage of healing occurs, radiologic union is present. Excess bone tissue is resorbed in the final stage of healing and union is complete. The bone gradually returns to its preinjury structure strength and shape. The osteoblasts and osteoclasts function normally in response to physical loading stress. The fracture hematoma stage occurs when the hematoma at the ends of the fragments becomes a semisolid blood clot. There is an unorganized network of bone composed of cartilage, osteoblasts, calcium, and phosphorus woven around fracture parts in the callus formation stage.

What should the occupational health nurse advise a patient whose job involves many hours of typing? a. Obtain a keyboard pad to support the wrist. b. Do stretching exercises before starting work. c. Wrap the wrists with compression bandages every morning. d. Avoid using nonsteroidal antiinflammatory drugs (NSAIDS).`

A Repetitive strain injuries caused by prolonged work at a keyboard can be prevented by using a pad to keep the wrists in a straight position. Stretching exercises during the day may be helpful, but these would not be needed before starting work. Use of a compression bandage is not needed, although a splint may be used for carpal tunnel syndrome. NSAIDs are appropriate to decrease swelling.

5) A client is seen for muscle spasms in the upper thigh. The nurse anticipates instructing the client on the use of which medication? 1. Nonsteroidal anti-inflammatory drugs 2. Muscle relaxants 3. Both nonsteroidal anti-inflammatory drugs and muscle relaxants 4. Nonsteroidal anti-inflammatory drugs, muscle relaxants, and narcotic pain relievers

Answer: 3 Explanation: When skeletal muscle relaxants are used in combination with nonsteroidal anti-inflammatories, pain relief is greater than when either agent is used alone. Page Ref: 361

The nurse should instruct a patient with a nondisplaced fractured left radius that the cast will need to remain in place for what amount of time? a. Two weeks b. At least six weeks c. Until swelling of the wrist has resolved d. Until x-rays show complete bony union.

B Bone healing starts immediately after the injury, but because ossification does not begin until 3 weeks after injury, the cast will need to be worn for at least 3 weeks. Complete union may take up to 1 year. Resolution of swelling does not indicate bone healing.

A patient who had a long leg cast applied this morning asks to crutch walk before dinner. Which statement explains why the nurse will decline the patient's request? "You must ambulate with a physical therapist for the first few days." "The cast is not dry yet, so it may be damaged while using crutches." "Rest, ice, compression, and elevation are in process to decrease pain." "Excess edema and complications are prevented when the leg is elevated for 24 hours."

"Excess edema and complications are prevented when the leg is elevated for 24 hours." Rationale: For the first 24 hours after a lower extremity cast is applied, the leg should be elevated on pillows above heart level to avoid excessive edema and compartment syndrome. RICE is used for soft tissue injuries, not with long leg casts.

1. Patient-Centered Care: A 72-year-old man tells the nurse that he cannot perform most of the physical activities he could do 5 years ago because of overall joint aches and pains. What can the nurse do to help the patient prevent further deconditioning and decrease the risk for developing musculoskeletal problems? a. Limit weight-bearing exercise to prevent stress on fragile bones and possible hip fractures. b. Tell the patient to avoid using canes and walkers because they increase dependence on ambulation aids. c. Teach the patient to increase his activity by climbing stairs in buildings and other environments with steps. d. Discuss the use of stretching and strengthening exercises to decrease aches and pain so that exercise can be maintained.

1. d. Almost all older adults have some degree of decreased muscle strength, joint stiffness, and pain with motion. Warming up before and strengthening exercises help decrease aches and pains. Musculoskeletal problems in the older adult can be prevented with appropriate strategies, especially exercise. Walkers and canes should be used as necessary to decrease stress on joints so activity can be maintained. Stair walking can create enough stress on fragile bones to cause a hip fracture.

For a patient who has had right hip arthroplasty, which nursing action can the nurse delegate to experienced unlicensed assistive personnel (UAP)? a. Reposition the patient every 1 to 2 hours. b. Assess for skin irritation on the patient's back. c. Teach the patient quadriceps-setting exercises. d. Determine the patient's pain intensity and tolerance.

A Repositioning of orthopedic patients is within the scope of practice of UAP after they have been trained and evaluated in this skill. The other actions should be done by licensed nursing staff members.

Which action should the nurse take to evaluate the effectiveness of Buck's traction for a patient who has an intracapsular fracture of the right femur? a. Assess for hip pain. b. Check for contractures. c. Palpate peripheral pulses. d. Monitor for hip dislocation.

A Buck's traction is used to reduce painful muscle spasm. Hip contractures and dislocation are unlikely to occur in this situation. The peripheral pulses will be assessed, but this does not help in evaluating the effectiveness of Buck's traction.

A patient with a right lower leg fracture will be discharged home with an external fixation device in place. Which statement should the nurse including in discharge teaching? a. "Check and clean the pin insertion sites daily." b. "Remove the external fixator for your shower." c. "Remain on bed rest until bone healing is complete." d. "Take prophylactic antibiotics until the fixator is removed."

A Pin insertion sites should be cleaned daily to decrease risk for infection at the site. An external fixator allows the patient to be out of bed and avoid the risks of prolonged immobility. The device is surgically placed and is not removed until the bone is stable. Prophylactic antibiotics are not routinely given during external fixator use.

A patient who has had open reduction and internal fixation (ORIF) of a hip fracture tells the nurse he is ready to get out of bed for the first time. Which action should the nurse take? a. Check the patient's prescribed weight-bearing status. b. Use a mechanical lift to transfer the patient to the chair. c. Decrease the pain medication before getting the patient up. d. Have the unlicensed assistive personnel (UAP) transfer the patient.

A The nurse should be familiar with the weight-bearing orders for the patient before attempting the transfer. Mechanical lifts are not typically needed after this surgery. Pain medications should be given because the movement is likely to be painful for the patient. The registered nurse (RN) should supervise the patient during the initial transfer to evaluate how well the patient is able to accomplish the transfer.

A patient who had open reduction and internal fixation (ORIF) of left lower leg fractures continues to report severe pain in the leg 15 minutes after receiving the prescribed IV morphine. The nurse determines pulses are faintly palpable and the foot is cool to the touch. Which action should the nurse take next? a. Notify the health care provider. b. Assess the incision for redness. c. Reposition the left leg on pillows. d. Check the patient's blood pressure

A The patient's clinical manifestations suggest compartment syndrome and delay in diagnosis and treatment may lead to severe functional impairment. The data do not suggest problems with blood pressure or infection. Elevation of the leg will decrease arterial flow and further reduce perfusion.

1. In which order will the nurse take these actions when caring for a patient in the emergency department with a right leg fracture after a motor vehicle accident? (Put a comma and a space between each answer choice [A, B, C, D, E, F].) a. Obtain x-rays. b. Check pedal pulses. c. Assess lung sounds. d. Take blood pressure. e. Apply splint to the leg. f. Administer tetanus prophylaxis.

ANS: C, D, B, E, A, F The initial actions should be to ensure that airway, breathing, and circulation are intact. This should be followed by checking the neurovascular status of the leg (before and after splint application). Application of a splint to immobilize the leg should be done before sending the patient for x-rays. The tetanus prophylaxis is the least urgent of the actions.

6. The nurse will instruct the patient with a fractured left radius that the cast will need to remain in place a. for several months. b. for at least 3 weeks. c. until swelling of the wrist has resolved. d. until x-rays show complete bony union.

ANS: B Bone healing starts immediately after the injury, but since ossification does not begin until 3 weeks postinjury, the cast will need to be worn for at least 3 weeks. Complete union may take up to a year. Resolution of swelling does not indicate bone healin

8. Which nursing intervention will be included in the plan of care after a patient with a right femur fracture has a hip spica cast applied? a. Avoid placing the patient in prone position. b. Ask the patient about abdominal discomfort. c. Discuss remaining on bed rest for several weeks. d. Use the cast support bar to reposition the patient.

ANS: B Assessment of bowel sounds, abdominal pain, and nausea and vomiting will detect the development of cast syndrome. To avoid breakage, the support bar should not be used for repositioning. After the cast dries, the patient can begin ambulating with the assistance of physical therapy personnel and may be turned to the prone position

45. When caring for a patient who is using Bucks traction after a hip fracture, which action can the nurse delegate to unlicensed assistive personnel (UAP)? a. Monitor the skin under the traction boot for redness. b. Ensure that the weight for the traction is off the floor. c. Check for intact sensation and movement in the affected leg. d. Offer reassurance that hip and leg pain are normal after hip fractur

ANS: B UAP can be responsible for maintaining the integrity of the traction once it has been established. Assessment of skin integrity and circulation should be done by the registered nurse (RN). UAP should notify the RN if the patient experiences hip and leg pain because pain and effectiveness of pain relief measures should be assessed by the RN

9. A patient has a long-arm plaster cast applied for immobilization of a fractured left radius. Until the cast has completely dried, the nurse should a. keep the left arm in dependent position. b. avoid handling the cast using fingertips. c. place gauze around the cast edge to pad any roughness. d. cover the cast with a small blanket to absorb the dampness.

ANS: B Until a plaster cast has dried, using the palms rather than the fingertips to handle the cast helps prevent creating protrusions inside the cast that could place pressure on the skin. The left arm should be elevated to prevent swelling. The edges of the cast may be petaled once the cast is dry, but padding the edges before that may cause the cast to be misshapen. The cast should not be covered until it is dry because heat builds up during drying

36. A 42-year-old patient is admitted to the emergency department with a left femur fracture. Which information obtained by the nurse is most important to report to the health care provider? a. Ecchymosis of the left thigh b. Complaints of severe thigh pain c. Slow capillary refill of the left foot d. Outward pointing toes on the left foot

ANS: C Prolonged capillary refill may indicate complications such as arterial damage or compartment syndrome. The other findings are typical with a left femur fracture

7. A 48-year-old patient with a comminuted fracture of the left femur has Bucks traction in place while waiting for surgery. To assess for pressure areas on the patients back and sacral area and to provide skin care, the nurse should a. loosen the traction and help the patient turn onto the unaffected side. b. place a pillow between the patients legs and turn gently to each side. c. turn the patient partially to each side with the assistance of another nurse. d. have the patient lift the buttocks by bending and pushing with the right leg.

ANS: D The patient can lift the buttocks off the bed by using the left leg without changing the right-leg alignment. Turning the patient will tend to move the leg out of alignment. Disconnecting the traction will interrupt the weight needed to immobilize and align the fractur

After being hospitalized for 3 days with a right femur fracture, a patient suddenly develops shortness of breath and tachypnea. The patient tells the nurse, "I feel like I am going to die!" Which action should the nurse take first? a. Stay with the patient and offer reassurance. b. Administer prescribed PRN O2 at 4 L/min. c. Check the patient's legs for swelling or tenderness. d. Notify the health care provider about the symptoms.

B The patient's clinical manifestations and history are consistent with a pulmonary embolism, and the nurse's first action should be to ensure adequate oxygenation. The nurse should offer reassurance to the patient but meeting the physiologic need for O2 is a higher priority. The health care provider should be notified after the O2 is started and pulse oximetry obtained concerning suspected fat embolism or venous thromboembolism.

A 60-yr-old patient had open reduction and internal fixation (ORIF) for an open, displaced tibial fracture, What should the nurse identify as the priority patient problem? a. Acute pain b. Risk for infection c. Activity intolerance d. Risk for constipation

B A patient having ORIF is at risk for problems such as wound infection and osteomyelitis. After ORIF, patients typically are mobilized starting the first postoperative day, so the problems caused by immobility are not as likely. Pain management is important, but the most important action is to prevent infection.

After change-of-shift report, which patient should the nurse assess first? a. Patient with a repaired mandibular fracture who is reporting facial pain. b. Patient with repaired right femoral shaft fracture who reports tightness in the calf. c. Patient with an unrepaired Colles' fracture who has right wrist swelling and deformity. d. Patient with an unrepaired intracapsular left hip fracture whose leg is externally rotated.

B Calf swelling after a femoral shaft fracture suggests possible DVT or compartment syndrome. The nurse should assess the patient rapidly and then notify the health care provider. The other patients have symptoms that are typical for their injuries but do not require immediate intervention.

Which information about a patient with a lumbar vertebral compression fracture should the nurse immediately report to the health care provider? a. Patient declines to be turned due to back pain. b. Patient has been incontinent of urine and stool. c. Patient reports lumbar area tenderness to palpation. d. Patient frequently uses oral corticosteroids to treat asthma.

B Changes in bowel or bladder function indicate possible spinal cord compression and should be reported immediately because surgical intervention may be needed. The other findings are also pertinent but are consistent with the patient's diagnosis and do not require immediate intervention.

After a motorcycle accident, a patient arrives in the emergency department with severe swelling of the left lower leg. Which action should the nurse take first? a. Elevate the leg on 2 pillows. b. Apply a compression bandage. c. Assess leg pulses and sensation. d. Place ice packs on the lower leg.

C The initial action by the nurse will be to assess circulation to the leg and observe for any evidence of injury such as fractures or dislocations. After the initial assessment, the other actions may be appropriate based on what is observed during the assessment.

A patient who is to have no weight bearing on the left leg is learning to use crutches. Which observation by the nurse indicates the patient can safely ambulate independently? a. The patient moves the right crutch with the right leg and then the left crutch with the left leg. b. The patient advances the left leg and both crutches together and then advances the right leg. c. The patient uses the bedside chair to assist in balance as needed when ambulating in the room. d. The patient keeps the padded area of the crutch firmly in the axillary area when ambulating.

B Patients are usually taught to move the crutches and the injured leg forward at the same time and then to move the unaffected leg. If the 2- or 4-point gait is to be used, the crutch and leg on opposite sides move forward, not the crutch and same-side leg. Patients are discouraged from using furniture to assist with ambulation. The patient is taught to place weight on the hands, not in the axilla, to avoid brachial plexus damage.

A patient who arrives at the emergency department with severe left knee pain is diagnosed with a patellar dislocation. What should be the nurse's initial focus for patient teaching? a. Use of a knee immobilizer b. Monitored anesthesia care c. Physical activity restrictions d. Performance of gentle knee flexion

B The first goal of interprofessional management is realignment of the knee to its original anatomic position, which will require anesthesia or monitored anesthesia care, formerly called conscious sedation. Immobilization, gentle range-of-motion exercises, and discussion about activity restrictions will be implemented after the patella is realigned.

The second day after admission with a fractured pelvis, a patient suddenly develops confusion. Which action should the nurse take first? a. Take the blood pressure. b. Check the O2 saturation. c. Assess patient orientation. d. Observe for facial asymmetry.

B The patient's history and clinical manifestations suggest a fat embolism. The most important assessment is oxygenation. The other actions are also appropriate but will be done after the nurse assesses O2 saturation.

The nurse is caring for a patient who is using Buck's traction after a hip fracture. Which action can the nurse delegate to experienced unlicensed assistive personnel (UAP)? a. Remove and reapply traction periodically. b. Ensure the weight for the traction is hanging freely. c. Monitor the skin under the traction boot for redness. d. Check for intact sensation and movement in the affected leg.

B UAP can be responsible for maintaining the integrity of the traction after it has been established. The RN should assess the extremity and assure manual traction is maintained if the traction device has to be removed and reapplied. Assessment of skin integrity and circulation should be done by the registered nurse (RN).

Which information should the nurse include in discharge instructions for a patient with comminuted left forearm fractures and a long-arm cast? a. Keep the left shoulder elevated on a pillow or cushion. b. Avoid nonsteroidal antiinflammatory drugs (NSAIDs). c. Call the health care provider for numbness of the hand. d. Keep the hand immobile to prevent soft tissue swelling.

C Increased swelling or numbness may indicate increased pressure at the injury, and the health care provider should be notified immediately to avoid damage to nerves and other tissues. The patient should be encouraged to move the joints above and below the cast to avoid stiffness. There is no need to elevate the shoulder, although the forearm should be elevated to reduce swelling. NSAIDs are appropriate to treat mild to moderate pain after a fracture.

A pedestrian who was hit by a car is admitted to the emergency department with possible right lower leg fractures. What initial action should the nurse take? a. Elevate the right leg. b. Splint the lower leg. c. Assess the pedal pulses. d. Verify tetanus immunization.

C The initial nursing action should be assessment of the neurovascular condition of the injured leg. After assessment, the nurse may need to splint and elevate the leg based on the assessment data. Information about tetanus immunizations should be obtained if there is an open wound.

A patient arrived at the emergency department after tripping over a rug and falling at home. Which finding should the nurse identify as most important to communicate to the health care provider? a. There is bruising at the shoulder area. b. The patient reports arm and shoulder pain. c. The right arm appears shorter than the left. d. There is decreased shoulder range of motion.

C A shorter limb after a fall indicates a possible dislocation, which is an orthopedic emergency. Bruising, pain, and decreased range of motion should also be reported, but these do not indicate emergent treatment is needed to preserve function.

A patient is being discharged 4 days after hip arthroplasty using the posterior approach. Which patient action requires intervention by the nurse? a. Using crutches with a swing-to gait b. Sitting upright on the edge of the bed c. Leaning over to pull on shoes and socks d. Bending over the sink while brushing teeth

C Leaning over would flex the hip at greater than 90 degrees and predispose the patient to hip dislocation. The other patient actions are appropriate and do not require any immediate action by the nurse to protect the patient.

When a patient arrives in the emergency department with a facial fracture, which action should the nurse take first? a. Assess for nasal bleeding and pain. b. Apply ice to the face to reduce swelling. c. Use a cervical collar to stabilize the spine. d. Check the patient's alertness and orientation.

C Patients who have facial fractures are at risk for cervical spine injury and should be treated as if they have a cervical spine injury until this is ruled out. The other actions are also necessary, but the most important action is to prevent cervical spine injury.

A patient is admitted to the emergency department with a left femur fracture. Which assessment finding by the nurse is most important to report to the health care provider? a. Bruising of the left thigh b. Reports of severe thigh pain c. Slow capillary refill of the left foot d. Outward pointing toes on the left foot

C Prolonged capillary refill may indicate complications such as compartment syndrome. The other findings are typical with a left femur fracture.

A patient with a complex pelvic fracture from a motor vehicle crash is on bed rest. Which assessment finding should indicate to the nurse a potential complication of the fracture? a. The patient states the pelvis feels unstable. b. The patient reports pelvic pain with palpation. c. Abdomen is distended, and bowel sounds are absent. d. Ecchymoses are visible across the abdomen and hips.

C The abdominal distention and absent bowel sounds may be due to complications of pelvic fractures such as paralytic ileus or hemorrhage or trauma to the bladder, urethra, or colon. Pelvic instability, abdominal pain with palpation, and abdominal bruising would be expected with this type of injury.

A patient who slipped and fell in the shower at home has a proximal left humerus fracture immobilized with a sling. Which intervention should the nurse include in the plan of care? a. Use surgical net dressing to hang the arm from an IV pole. b. Immobilize the fingers of the left hand with gauze dressings. c. Assess the left axilla and change absorbent dressings as needed. d. Assist the patient in passive range of motion (ROM) for the right arm.

C The axilla can become excoriated when a sling is used to support the arm, and the nurse should check the axilla and apply absorbent dressings to prevent this. A patient with a sling would not have traction applied by hanging. The patient will be encouraged to move the fingers on the injured arm to maintain function and to help decrease swelling. The patient will do active ROM on the uninjured side.

The nurse is caring for a patient placed in Buck's traction before open reduction and internal fixation of a left hip fracture. Which care can be delegated to the LPN/VN? Assess skin integrity around the traction boot. Determine correct body alignment to enhance traction. Remove weights from traction when turning the patient. Monitor pain intensity and administer prescribed analgesics.

Correct Answer: Monitor pain intensity and administer prescribed analgesics. Rationale: The LPN/VN can monitor pain intensity and administer analgesics. Assessment of skin integrity and determining correct alignment to enhance traction are within the RN scope of practice. Removing weights from the traction should not be delegated or done. Removal of weights can cause muscle spasms and bone misalignment and should not be delegated or done.

Which action should the nurse include in the plan of care for a patient who had a cemented right total knee arthroplasty? a. Avoid extension of the right knee beyond 120 degrees. b. Use a compression bandage to keep the right knee flexed. c. Teach about the need to avoid weight bearing for 4 weeks. d. Start progressive knee exercises to obtain 90-degree flexion.

D After knee arthroplasty, active or passive flexion exercises are used to obtain a 90-degree flexion of the knee. The goal for extension of the knee will be 180 degrees. A compression bandage is used to hold the knee in an extended position after surgery. Protected weight bearing is typically not ordered after this procedure.

A young adult arrives in the emergency department with ankle swelling and severe pain after twisting an ankle playing basketball. Which prescribed action will the nurse implement first? a. Send the patient for ankle x-rays. b. Administer naproxen (Naprosyn). c. Give acetaminophen with codeine. d. Wrap the ankle and apply an ice pack.

D Immediate care after a sprain or strain injury includes application of cold and use of compression to minimize swelling. The other actions should be taken after the ankle is wrapped with a compression bandage and ice is applied.

A patient is to be discharged from the hospital 4 days after insertion of a femoral head prosthesis using a posterior approach. Which patient statement to the nurse indicates that additional teaching is needed? a. "I should not cross my legs while sitting." b. "I will use a toilet elevator on the toilet seat." c. "I will have someone else put on my shoes and socks." d. "I can sleep in any position that is comfortable for me."

D The patient needs to sleep in a position that prevents excessive internal rotation or flexion of the hip. The other patient statements indicate the patient has understood the teaching.

A factory line worker has repetitive strain syndrome in the left elbow. What topic should the nurse plan to include in patient teaching? a. Surgical options b. Elbow injections c. Wearing a left wrist splint d. Modifying arm movements

D Treatment for repetitive strain syndrome includes changing the ergonomics of the activity. Elbow injections and surgery are not initial options for this type of injury. A wrist splint might be used for hand or wrist pain.

Priority Decision: Before re-positioning the patient on the side after a lumbar laminectomy, what should be the nurse's first action a. raise the head of the bed 30 degrees b. Have the patient flex the knees and hips c. Place a pillow between the patient's legs d. Have the patient grasp the side rail on the opposite side of the bed

c After spinal surgery, patients are logrolled to maintain straight alignment of the spine. This involves turning the patient with a pillow between the legs and moving the body as a unit. The head of the bed is usually kept flat and the legs are extended.

30. What should the nurse include in discharge instructions for the patient after a hip prosthesis with a posterior approach? a. Restrict walking for 2 to 3 months.b. Take a bath rather than a shower to prevent falling.c. Keep the leg internally rotated while sitting and standing.d. Have a family member help the patient put on shoes and socks.

30. d. Having someone else put the patient's socks and shoes on for patients with hip prostheses with a posterior approach will protect the patient from extreme flexion, adduction, or internal rotation for at least 6 weeks to prevent dislocation of the prosthesis. Gradual weight bearing on the limb is allowed and ambulation should be encouraged. The leg should be not be internally rotated but kept in a neutral position.

38. When the nursing student asks the registered nurse (RN) what an arthroplasty is, what is the best description the RN can give the student? a. Surgical fusion of a joint to relieve painb. Correction of bone deformity by removal of a wedge or slice of bone c. Reconstruction or replacement of a joint to relieve pain and correct deformityd. Used in rheumatoid arthritis to remove the tissue involved in joint destruction

38. c. An arthroplasty is reconstruction or replacement of a joint to relieve pain and correct deformity, especially with osteoarthritis, RA, avascular necrosis, congenital deformity, or dislocation. Arthrodesis is the surgical fusion of a joint to relieve pain. An osteotomy removes a wedge of bone to correct a bone deformity. Synovectomy is used in RA to remove the tissue involved in joint destruction.

39. A patient had a right total hip arthroplasty with a cemented prosthesis for treatment of severe osteoarthritis of the hip. What activity should the nurse include on the patient's first postoperative day? a. Transfer from the bed to the chair twice a day only b. Only turning from the back to both sides every 2 hours c. Crutch walking with non-weight bearing on the operative leg d. Ambulation with a walker and limited weight bearing on the right leg

39. d. Physical therapy is initiated on the first postoperative day with ambulation using a walker and limited weight bearing for a patient with a cemented prosthesis and non- weight-bearing on the operative side for an uncemented prosthesis. In addition, the patient sits in the chair at least twice a day and is turned to the back and unaffected side with the operative leg supported. Crutches would be difficult to use the first postoperative day.

10. A patient is brought to the emergency department (ED) with an injured lower left leg following a fall while rock climbing. The nurse identifies the presence of a fracture based on what cardinal sign of fracture? a. Muscle spasms b. Obvious deformity c. Edema and swelling d. Pain and tenderness

10. b. Deformity is the cardinal sign of fracture but may not be apparent in all fractures. Other supporting signs include edema and swelling, localized pain and tenderness, muscle spasm, ecchymosis, loss of function, crepitation, and an inability to bear weight.

11. A patient with a fractured femur develops the complication of malunion. What does the nurse recognize has happened to the patient? a. The fracture heals in an unsatisfactory position. b. The fracture fails to heal properly despite treatment. c. Fracture healing progresses more slowly than expected. d. Loss of bone substances occurs as a result of immobilization.

11. a. A malunion occurs when the bone heals in the expected time but in an unsatisfactory position, possibly resulting in deformity or dysfunction. Nonunion occurs when the fracture fails to heal completely despite treatment. Delayed union is healing of the fracture at a slower rate than expected. In posttraumatic osteoporosis, the loss of bone substances occurs as a result of immobilization.

12. What is a disadvantage of open reduction and internal fixation (ORIF) of a fracture compared to closed reduction? a. Infectionb. Skin irritationc. Nerve impairmentd. Complications of immobility

12. a. Open reduction involves use of a surgical incision to visualize the fracture and correct bone alignment. The main disadvantage is the risk of infection. Anesthesia complications are also possible. Preexisting medical conditions may have impact on surgical success. Skin irritation and nerve impairment are most likely with skin traction. Prolonged immobility is possible with skeletal traction.

13. A young patient with a fractured femur has a hip spica cast applied. What is the most important action the nurse should take while the cast is drying? a. Elevate the legs above the level of the heart for 24 hours. b. Turn the patient to both sides and prone to supine every 2 hours. c. Cover the cast with a light blanket to avoid chilling from evaporation. d. Assess the patient frequently for abdominal pain, nausea, and vomiting.

13. d. Abdominal pain or pressure, nausea, and vomiting are signs of cast syndrome that occur when hip spica casts or body jacket braces cause compression of the superior mesenteric artery against the duodenum because of swelling or tight application. The cast may have to be split or removed, and the HCP should be notified. Elevation is not indicated for a spica cast. The patient with a spica cast should not be placed in the prone position during the initial drying stage because the cast may break because it is so large and heavy. A cast should never be covered with a blanket because heat builds up in the cast and may cause tissue damage.

14. Priority Decision: A patient is admitted with an open fracture of the tibia after a bicycle accident. What question should the nurse ask when assessing the patient? a. Any previous injuries to the leg b. The status of tetanus immunization c. The use of antibiotics in the last month d. Whether the injury was exposed to dirt or gravel

14. b. Tetanus prevention is always indicated if the patient has not been immunized or does not have a current booster. Infection is the greatest risk with an open fracture, and all open fractures are considered contaminated. Although prophylactic antibiotics are used in management of open fractures, neither recent antibiotic therapy nor previous injury to the site is relevant. Dirt or gravel contamination will be evident on physical assessment.

15. Priority Decision: A patient who fell in the bathroom of the hospital room reports pain in the upper right arm and elbow. Which action should the nurse take first in managing a possible fracture before splinting the injury? a. Elevate the arm b. Apply ice to the site c. Notify the health care provider d. Perform a neurovascular check below the injury

15. d. Sensation, motor function, and pain distal to the injury should be checked before and after splinting to assess for nerve damage. Document results to avoid doubts about whether a problem discovered later was missed during the original examination or was caused by the treatment. Peripheral vascular assessment is needed and the HCP is notified. Elevation

16. How should the nurse assess the neurologic status of the patient with a fractured humerus? a. Have the patient evert, invert, dorsiflex, and plantar flex the foot. b. Assess the location, quality, and intensity of pain below the site of the injury. c. Have the patient abduct the fingers, oppose the thumb and fingers, and flex and extend the wrist. d. Assess the color, temperature, capillary refill, peripheral pulses, and edema in the extremity.

16. c. Neurologic assessment includes evaluation of sensation, motor function, and pain in the upper extremity. Ask the patient to abduct the fingers (ulnar nerve), oppose the thumb and small fingers (median nerve), and flex and extend the wrist (or fingers if in a cast) (radial nerve). The nurse will assess pain and sensory function in the fingers. Evaluation of the feet would occur in lower extremity injuries. Assessment of color, temperature, capillary refill, peripheral pulses, and edema evaluates vascular condition.

17. Patient-Centered Care: A patient is discharged from the outpatient clinic after application of a synthetic fiberglass long arm cast for a fractured ulna. Before discharge, what instruction should the nurse provide to the patient? a. Never get the cast wet. b. Move the shoulder and fingers frequently. c. Place tape petals around the edges of the cast when it is dry. d. Use a sling to support the arm at waist level for the first 48 hours.

17. b. A patient with any type of cast should exercise the joints above and below the cast frequently. Moving the fingers will improve circulation and help prevent edema. Unlike plaster casts, thermoplastic resin or fiberglass casts are relatively waterproof. If they become wet, they can be dried with a hair dryer on low setting. Tape petals are used on plaster casts to protect the edges from breaking and crumbling but are not needed for synthetic casts. After the cast is applied, the extremity should be elevated at the level of the heart to promote venous return. Ice may be used to prevent edema.

18. Patient-Centered Care: A patient with a fractured tibia accompanied by extensive soft tissue damage initially has a splint applied and held in place with an elastic bandage. What early sign would alert the nurse that the patient is developing compartment syndrome? a. Paralysis of the toes b. Absence of peripheral pulses c. Distal pain unrelieved by opioid analgesics d. Skin over the injury site is blanched when the bandage is removed

18. c. Pain that is distal to the injury and unrelieved by opioid analgesics is the earliest sign of compartment syndrome; paresthesia is also an early sign. Paralysis and absence of peripheral pulses will eventually occur if there is no treatment, but these are late signs that often appear after permanent damage has occurred. The overlying skin may appear normal because the surface vessels are not occluded.

19. If surgery is needed, which procedure would the nurse first prepare the patient for in the presence of compartment syndrome? a. Fasciotomyb. Amputationc. Internal fixationd. Release of tendons

19. a. Soft tissue edema in the area of the injury may cause increased pressure within the closed tissue compartments formed by the nonelastic fascia, causing compartment syndrome. If symptoms occur, surgical incision of fascia may be needed (fasciotomy). Amputation is usually needed only if the limb becomes septic because of untreated compartment syndrome.

2. The patient asks, "What does the doctor mean when he says that I have an avulsion fracture in my leg? I thought I had a sprain!" What is the best response by the nurse? a. "It is a fracture with more than 2 fragments."b. "It means that a ligament pulled a bone fragment loose."c. "The line of the fracture is twisted along the shaft of the bone."d. "The line of the fracture is at right angles to the longitudinal axis of the bone."

2. b. An avulsion fracture occurs when a ligament pulls a bone fragment loose. The pain is similar to a sprain. A fracture with 2 or more fragments is a comminuted fracture. A spiral fracture is twisted around a bone shaft. A transverse fracture occurs when the line of fracture is at right angles to the longitudinal axis.

20. Which type of fracture can cause radial nerve and brachial artery damage and is reduced with a hanging arm cast? a. Fractured tibia b. Colles' fracture c. Fractured humerus d. Femoral shaft fracture

20. c. A fractured humerus may cause radial nerve and brachial artery damage. It may be reduced nonsurgically with a hanging arm cast. A fractured tibia and femur are in the leg. The Colles' fracture is in the wrist and manifests with pronounced swelling and obvious deformity of the wrist. It is treated with closed manipulation and immobilization.

23. In a patient with a stable vertebral fracture, what should the nurse teach the patient? a. Remain on bed rest until the pain is gone. b. Logroll to keep the spine straight when turning. c. How to use bone cement to correct the problem. d. Take as much analgesic as needed to relieve the pain.

23. b. The spine should be kept straight by turning the shoulders and hips together (logrolling). This keeps the spine in good alignment until union has been accomplished. Bed rest may be required for a short time but not until the pain is gone. Bone cement is used by the surgeon to stabilize vertebral compression fractures. Analgesics should be taken only as ordered. If they do not manage the pain, the HCP should be notified.

24. When is a fat embolism most likely to occur? a. 24 to 48 hours after a fractured tibia b. 36 to 72 hours after a skull fracture c. 4 to 5 days after a fractured femur d. 5 to 6 days after a pelvic fracture

24. a. Initial manifestations of fat embolism usually occur 24 to 48 hours after injury. They are associated with fractures of long bones and multiple fractures related to pelvic injuries, including fractures of the femur, tibia, ribs, and pelvis. Venous thromboemboli (VTEs) tend to form later after injury of the extremities and pelvis.

25. What assessment findings distinguish a fat embolism from a pulmonary embolism in a patient with a fracture? a. Tachycardia and dyspnea b. A sudden onset of chest pain c. Petechiae around the neck and upper chest d. Electrocardiographic (ECG) changes and decreased partial pressure of oxygen in arterial blood (PaO2)

25. c. Patients with fractures are at risk for both fat embolism and pulmonary embolism from VTE, but there is a difference in the time of occurrence. Fat embolism occurs shortly after the injury and thrombotic embolism occurring several days after immobilization. They both may cause pulmonary symptoms of chest pain, tachypnea, dyspnea, apprehension, tachycardia, and cyanosis. However, only fat embolism may cause petechiae found around the neck, anterior chest wall, axilla, buccal membrane of the mouth, and conjunctiva of the eye.

26. Which kind of hip fracture is usually repaired with a hip prosthesis? a. Intracapsular b. Extracapsularc. Subtrochanteric d. Intertrochanteric

26. a. A hip prosthesis is usually used for intracapsular fractures. The other options are all for extracapsular fractures.

27. An older adult woman is admitted to the ED after falling at home. The nurse cautions her not to put weight on the leg after finding what in the patient assessment? a. Inability to move the toes and ankle b. Edema of the thigh extending to the knee c. Internal rotation of the leg with groin pain d. Shortening and external rotation of the leg

27. d. The classic signs of a hip fracture are external rotation and shortening of the leg accompanied by severe pain at the fracture site. Additional injury could be caused by weight bearing on the extremity. The patient may not be able to move the hip or the knee, but movement in the ankle and toes is not affected.

28. A patient with an extracapsular hip fracture is admitted to the orthopedic unit and placed in Buck's traction. How should the nurse explain the purpose of the traction to the patient? a. Pulls bone fragments back into alignment b. Immobilizes the leg until healing is complete c. Reduces pain and muscle spasms before surgery d. Prevents damage to the blood vessels at the fracture site

28. c. Although surgical repair is the preferred method of managing intracapsular and extracapsular hip fractures, patients may be treated initially with skin traction (e.g., Buck's traction) to relieve the painful muscle spasms before surgery is performed. Prolonged traction would be required to reduce the fracture or immobilize it for healing, creating a very high risk for complications of immobility.

29. A patient with a fractured right hip has an anterior ORIF of the fracture. What should the nurse plan to do postoperatively? a. Get the patient up to the chair on the first postoperative day. b. Ambulate the patient with partial weight bearing by discharge. c. Keep the leg abductor pillow on the patient even when bathing. d. Position the patient only on the back and the nonoperative side.

29. a. Because the fracture site is internally fixed with pins or plates, the fracture site is stable and the patient is moved from the bed to the chair on the first postoperative day. Ambulation begins on the first or second postoperative day without weight bearing on the affected leg. Weight bearing on the affected extremity is usually restricted for 6 to 12 weeks until adequate healing is evident on x-ray. Abductor pillows are used for patients who have total hip replacements by the posterior surgical approach. The patient may be positioned on the operative side following internal fixation as prescribed by the HCP.

4. The patient works on a computer 8 hours each day. What kind of injury would the nurse assess for in this patient? a. Meniscus injury b. Rotator cuff injury c. Radial-ulnar fracture d. Carpal tunnel syndrome

4. d. Carpal tunnel syndrome would be expected related to continuous wrist movements. Injuries of the meniscus (fibrocartilage in the knee) are common with athletes. Radial- ulnar fractures are seen with great force, such as a fall or a car accident. Rotator cuff injuries occur with sudden adduction forces applied to the cuff while the arm is held in abduction. They are often seen with repetitive overhead motions.

40. When positioning the patient after a total hip arthroplasty with a posterior approach, it is important that the nurse maintain the affected extremity in what position? a. Adduction and flexion b. Abduction and extension c. Abduction and internal rotation d. Adduction and external rotation

40. b. During hospitalization, after a total hip arthroplasty with a posterior approach, an abduction pillow is placed between the legs to maintain abduction and the leg is extended. Extremes of internal rotation, adduction, and 90-degree flexion of the hip must be avoided for 4 to 6 weeks postoperatively to prevent dislocation of the prosthesis.

41. After a total knee arthroplasty, a patient has a continuous passive motion (CPM) machine for the affected joint. What should the nurse explain to the patient is the purpose of this device? a. To relieve edema and pain at the incision siteb. To promote early joint mobility and increase knee flexionc. To prevent venous stasis and the formation of a deep venous thrombosisd. To improve arterial circulation to the affected extremity to promote healing

41. b. Continuous passive motion (CPM) machines may be used after knee surgery to promote early joint mobility. Because joint dislocation is not a problem with knee replacements, early ambulation to prevent deep vein thrombosis (DVT) and improve circulation, exercise with straight leg raises, and gentle ROM maybe encouraged postoperatively.

42. Priority Decision: A patient with severe ulnar deviation of the hands undergoes an arthroplasty with reconstruction and replacement of finger joints. Postoperatively, what is it most important for the nurse to do? a. Position the fingers lower than the elbow at all times. b. Perform neurovascular assessments of the fingers every 2 to 4 hours. c. Encourage the patient to gently flex, extend, abduct, and adduct the fingers every 4 hours. d. Remind the patient that function of the hands is more important than cosmetic appearance.

42. b. Neurovascular checks following surgery are essential to detect compromised neurologic and vascular function caused by trauma or edema. Postoperatively, the hands are elevated with a bulky dressing in place. When the dressing is removed, a guided splinting program is started. After discharge, exercises are done 3 to 4 times each day. Before surgery, the patient is taught that the goal of the surgery is to restore function related to grasp, pinch, stability, and strength and the hands, not cosmetic appearance.

43. Priority Decision: After change-of-shift handoff, which patient should the nurse assess first? a. A 58-year-old male reporting phantom pain and requesting an analgesic b. A 72-year-old male being transferred to a skilled nursing unit after repair of a hip fracture c. A 25-year-old female in left leg skeletal traction asking for the weights to be lifted for a few minutes d. A 68-year-old male with a new lower leg cast reporting the cast is too tightand he cannot feel his toes

43. d. The patient with a tight cast may be at risk for neurovascular compromise (impaired circulation and peripheral nerve damage) and should be assessed first. The other patients should be seen as soon as possible. Providing analgesia for the patient with phantom pain would be the next priority. The patient in skeletal traction needs explanation of the purpose and functioning of the traction. She may need analgesia or muscle relaxants to help tolerate the traction. Checking on the patient being transferred would include reassurance and paperwork completion.

6. Application of RICE (rest, ice, compression, elevation) is indicated for initial management of which type of injury? a. Muscle spasms b. Sprains and strains c. Repetitive strain injury d. Dislocations and subluxations

6. b. Rest, ice, compression, and elevation (RICE) are indicated to decrease edema resulting from sprains and some strains. Muscle spasms are usually treated with heat application and massage. Repetitive strain injuries require cessation of the precipitating activity and physical therapy. Dislocations or subluxations require immediate reduction and immobilization to prevent vascular impairment and bone cell death.

7. What should be included in the management during the first 48 hours after an acute soft tissue injury of the ankle (select all that apply)? a. Use of elastic wrap b. Initial immobilization and rest c. Elevation of ankle above the heart d. Alternating the use of heat and cold e. Administration of antiinflammatory drugs

7. a, b, c, e. Consider the principle of RICE. Rest: movement should be restricted. Ice: cold should be used to promote vasoconstriction and reduce edema. Compression: helps decrease swelling. Elevate: the extremity above the level of the heart. Mild nonsteroidal antiinflammatory drugs (NSAIDs) may be needed to manage pain. Warm, moist compresses may be used after 48 hours for 20 to 30 minutes at a time to reduce swelling and provide comfort.

1. In which order should the nurse implement interventions prescribed for a patient admitted with acute osteomyelitis who has a temperature of 101.2° F? (Put a comma and a space between each answer choice [A, B, C, D].) a. Obtain blood cultures from two sites. b. Administer dose of gentamicin 60 mg IV. c. Send to radiology for computed tomography (CT) scan of right leg. d. Administer acetaminophen (Tylenol) now and every 4 hours PRN for fever.

ANS: A, B, D, C The highest treatment priority for osteomyelitis is initiation of antibiotic therapy, but cultures should be obtained before administration of antibiotics. Addressing the discomfort of the fever is the next highest priority. Because the purpose of the CT scan is to determine the extent of the infection, it can be done last.

21. A nurse who works on the orthopedic unit has just received change-of-shift report. Which patient should the nurse assess first? a. Patient who reports foot pain after hammertoe surgery. b. Patient who has not voided 8 hours after a laminectomy. c. Patient with low back pain and a positive straight-leg-raise test. d. Patient with osteomyelitis who has a temperature of 100.5° F (38.1° C).

ANS: B Difficulty in voiding may indicate damage to the spinal nerves and should be assessed and reported to the surgeon immediately. The information about the other patients is consistent with their diagnoses. The nurse will need to assess them as quickly as possible, but the information about them does not indicate a need for immediate intervention.

7. A patient whose employment requires frequent lifting has a history of chronic back pain. After the nurse has taught the patient about correct body mechanics, which patient statement indicates the teaching has been effective? a. "I will keep my back straight when I lift above than my waist." b. "I will begin doing exercises to strengthen and support my back." c. "I will tell my boss I need a job where I can stay seated at a desk." d. "I can sleep with my hips and knees extended to prevent back strain."

ANS: B Exercises can help strengthen the muscles that support the back. Flexion of the hips and knees places less strain on the back than keeping these joints extended. Sitting for prolonged periods can aggravate back pain. Modification in the way the patient lifts boxes is needed, but the patient should not lift above the level of the elbows.

1. A patient with acute osteomyelitis of the left femur is hospitalized for regional antibiotic irrigation. Which intervention should the nurse include in the initial plan of care? a. Quadriceps-setting exercises b. Immobilization of the left leg c. Positioning the left leg in flexion d. Assisted weight-bearing ambulation

ANS: B Immobilization of the affected leg helps to decrease pain and reduce the risk for pathologic fracture. Weight-bearing exercise increases the risk for pathologic fractures. Flexion of the affected limb is avoided to prevent contractures.

2. A patient is being discharged after 1 week of IV antibiotic therapy for acute osteomyelitis in the right leg. Which information should the nurse include in the discharge teaching? a. How to apply warm packs to the leg to reduce pain b. How to monitor and care for a long-term IV catheter c. The need for daily aerobic exercise to help maintain muscle strength d. The reason for taking oral antibiotics for 7 to 10 days after discharge

ANS: B The patient will be taking IV antibiotics for several months. The patient will need to recognize signs of infection at the IV site and know how to care for the catheter during daily activities such as bathing. IV antibiotics rather than oral antibiotics are used for acute osteomyelitis. Patients are instructed to avoid exercise and heat application because these will increase swelling and the risk for spreading infection.

8. Which action should the nurse take when repositioning the patient who has just had a laminectomy and discectomy? a. Instruct the patient to move the legs before turning the rest of the body. b. Place a pillow between the patient's legs and turn the entire body as a unit. c. Have the patient turn by grasping the side rails and pulling the shoulders over. d. Turn the patient's head and shoulders first, followed by the hips, legs, and feet.

ANS: B The spine should be kept in correct alignment after laminectomy. The other positions will create misalignment of the spine.

3. A patient is receiving IV antibiotics at home to treat chronic osteomyelitis of the left femur. Which statement by the patient should indicate to the nurse the need for additional teaching related to health maintenance? a. "I'm frustrated with this endless treatment!" b. "I will take my oral temperature twice a day." c. "I think my left foot is starting to droop down." d. "I use crutches to avoid weight bearing on the left leg."

ANS: C Footdrop is an indication that the foot is not being supported in a neutral position by a splint. Using crutches and monitoring the oral temperature are appropriate self-care activities. Frustration with the length of treatment is not an indicator of ineffective health maintenance of the osteomyelitis.

18. Which action included in the care of a patient after laminectomy can the nurse delegate to experienced unlicensed assistive personnel (UAP)? a. Check ability to plantar and dorsiflex the foot. b. Determine the patient's readiness to ambulate. c. Log roll the patient from side to side every 2 hours. d. Ask about pain management with the patient-controlled analgesia (PCA).

ANS: C Repositioning a patient is included in the education and scope of practice of UAP, and experienced UAP will be familiar with how to maintain alignment in the postoperative patient. Evaluation of the effectiveness of pain medications, assessment of neurologic function, and evaluation of a patient's readiness to ambulate after surgery require higher level nursing education and scope of practice.

6. What should the nurse include in the teaching plan for ae patient who has acute low back pain and muscle spasms? a. Keep both feet flat on the floor when prolonged standing is required. b. Twist gently from side to side to maintain range of motion in the spine. c. Keep the head elevated slightly and flex the knees when resting in bed. d. Avoid the use of cold packs because they will exacerbate the muscle spasms.

ANS: C Resting with the head elevated and knees flexed will reduce the strain on the back and decrease muscle spasms. Twisting from side to side will increase tension on the lumbar area. Prolonged standing will cause strain on the lumbar spine, even with both feet flat on the floor. Alternate application of cold and heat should be used to decrease pain.

16. After laminectomy with a spinal fusion to treat a herniated disc, a patient reports numbness and tingling of the right lower leg. What action should the nurse take? a. Elevate the right leg on two pillows. b. Obtain vital signs for indication of hemorrhage. c. Review the preoperative assessment data in the health record. d. Turn the patient to the left to relieve pressure on the right leg.

ANS: C The postoperative movement and sensation of the extremities should be unchanged (or improved) from the preoperative assessment. If the numbness and tingling are new, this information should be immediately reported to the surgeon. Numbness and tingling are not symptoms associated with hemorrhage at the site. Turning the patient or elevating the leg will not relieve the numbness.

2. Which information should the nurse include when teaching a patient with acute low back pain? (Select all that apply.) a. Sleep in a prone position with the legs extended. b. Keep the knees straight when leaning forward to pick something up. c. Expect symptoms of acute low back pain to improve in a few weeks. d. Avoid activities that require twisting of the back or prolonged sitting. e. Use ibuprofen (Motrin, Advil) or acetaminophen (Tylenol) to relieve pain.

ANS: C, D, E Acute back pain usually starts to improve within 2 weeks. In the meantime, the patient should use medications such as nonsteroidal antiinflammatory drugs (NSAIDs) or acetaminophen to manage pain and avoid activities that stress the back. Sleeping in a prone position and keeping the knees straight when leaning forward will place stress on the back and should be avoided.

15. A patient has had surgical reduction of an open fracture of the right radius. Which assessment findings should the nurse report immediately to the health care provider? a. Serous wound drainage b. Right arm muscle spasms c. Pain with right arm movement d. Temperature 101.4° F (38.6° C)

ANS: D An elevated temperature suggests possible osteomyelitis. The other clinical manifestations are typical after a repair of an open fracture.

14. Which action should the nurse take before administering gentamicin (Garamycin) to a patient with acute osteomyelitis? a. Ask the patient about any nausea. b. Obtain the patient's oral temperature. c. Change the prescribed wet-to-dry dressings. d. Review the patient's serum creatinine results.

ANS: D Gentamicin is nephrotoxic and can cause renal failure as reflected in the patient's serum creatinine. Monitoring the patient's temperature before gentamicin administration is not necessary. Nausea is not a common side effect of IV gentamicin. There is no need to change the dressing before gentamicin administration.

9. Which statement by a patient with discomfort from a bunion indicates to the nurse that more teaching is needed? a. "I will give away my high-heeled shoes." b. "I can take ibuprofen (Motrin) if I need it." c. "I will use the bunion pad to cushion the area." d. "I can only wear sandals, no closed-toe shoes."

ANS: D The patient can wear shoes that have a wide forefoot (toe box). The other patient statements indicate the teaching has been effective.

What should the nurse include when teaching older adults at a community recreation center about ways to prevent fractures? a. Tack down scatter rugs on the floor in the home. b. Expect most falls to happen outside the home in the yard. c. Buy shoes that provide good support and are comfortable to wear. d. Get instruction in range-of-motion exercises from a physical therapist.

C Comfortable shoes with good support will help decrease the risk for falls. Scatter rugs should be eliminated, not just tacked down. Activities of daily living provide range-of-motion exercise; these do not need to be taught by a physical therapist. Falls inside the home are responsible for many injuries.

Which discharge instruction should the emergency department nurse include for a patient with a sprained ankle? a. Keep the ankle loosely wrapped with gauze. b. Apply a heating pad to reduce muscle spasms. c. Use pillows to elevate the ankle above the heart. d. Gently move the ankle through the range of motion.

C Elevation of the leg will reduce swelling and pain. Compression bandages are used to decrease swelling. For the first 24 to 48 hours, cold packs are used to reduce swelling. The ankle should be rested and kept immobile to prevent further swelling or injury.

Which patient statement indicates understanding of the nurse's teaching about a new short-arm synthetic cast? a. "I can remove the cast in 4 weeks using industrial scissors." b. "I should avoid moving my fingers until the cast is removed." c. "I will apply an ice pack to the cast over the fracture site off and on for 24 hours." d. "I can use a cotton-tipped applicator to rub lotion on any dry areas under the cast."

C Ice application for the first 24 hours after a fracture will help reduce swelling and can be placed over the cast. The cast is typically removed in the outpatient setting. The patient should be encouraged to move the joints above and below the cast. Patients should not insert objects inside the cast.

The nurse is caring for a patient who has a pelvic fracture and an external fixation device. How should the nurse perform assessment of pressure areas and provide skin care to the patient's back and sacrum? a. Ask the patient to turn to the side independently. b. Defer back assessment until the patient is ambulatory. c. Have the patient lift the back and buttocks using a trapeze. d. Roll the patient over to the side by pushing on the patient's hips.

C The patient can lift the back slightly off the bed by using a trapeze. The patient may find it very difficult to turn to the side without assistance while in a fixator device. Delaying assessment and skin care may put the patient at risk for an undetected pressure injury. Pushing on the patient's hips may cause additional injury.

The nurse has reviewed proper body mechanics with a patient who has a history of low back pain caused by a herniated lumbar disc. Which patient statement indicates a need for further teaching? "I should sleep on my side or back with my hips and knees bent." "I should exercise at least 15 minutes every morning and evening." "I should pick up items by leaning forward without bending my knees." "I should try to keep one foot on a stool whenever I have to stand for a period of time."

Correct Answer: "I should pick up items by leaning forward without bending my knees." Rationale: The patient should avoid leaning forward without bending the knees. Bending the knees helps to prevent lower back strain and is part of proper body mechanics for lifting. Sleeping on the side or back with hips and knees bent and standing with a foot on a stool will decrease lower back strain. Back strengthening exercises are done twice a day once symptoms subside.

The nurse is planning health promotion teaching for a 45-yr-old patient with asthma who has low back pain from herniated lumbar disc. What activity would the nurse include in an individualized exercise plan for the patient? Yoga Walking Calisthenics Weightlifting

Correct Answer: Walking Rationale: The patient would benefit from an aerobic exercise that considers the patient's health status and fits the patient's lifestyle. The best exercise is walking, which builds strength in the back and leg muscles without putting undue pressure or strain on the spine. If the patient has exercise-induced asthma, the nurse would recommend use of a rescue inhaler prior to exercise. Yoga, calisthenics, and weightlifting would all put pressure on or strain the spine.

The nurse provides instructions to a 30-yr-old office worker who has low back pain. Which statement indicate additional patient teaching is required? "Switching between hot and cold packs may relieve pain and stiffness." "Acupuncture to the lower back would cause irreparable nerve damage." "Smoking may aggravate back pain by decreasing blood flow to the spine." "Sleeping on my side with knees and hips bent reduces stress on my back."

Correct Answer: "Acupuncture to the lower back would cause irreparable nerve damage." Rationale: Acupuncture is a safe therapy when the practitioner has been appropriately trained. Very fine needles are inserted into the skin to stimulate specific anatomic points in the body for therapeutic purposes.

The nurse is admitting a patient who reports the new onset of lower back pain. To distinguish between the pain of a lumbar herniated disc from other causes, what is the best question for the nurse to ask the patient? "Is the pain worse in the morning or in the evening?" "Is the pain sharp and stabbing or burning and aching?" "Does the pain radiate down the buttock or into the leg?" "Is the pain totally relieved by acetaminophen (Tylenol)?"

Correct Answer: "Does the pain radiate down the buttock or into the leg?" Rationale: Lower back pain associated with a herniated lumbar disc is accompanied by radiation along the sciatic nerve. It is often described as traveling through the buttock to the posterior thigh or down the leg. This is because the herniated disc causes compression on spinal nerves as they exit the spinal column. Time of occurrence, type of pain, and pain relief questions do not elicit differentiating data.

A patient with acute osteomyelitis asks the nurse how this problem will be treated initially. Which response by the nurse is mostappropriate? "IV antibiotics are usually required for several weeks." "Oral antibiotics are often required for several months." "Surgery is almost always necessary to remove the dead tissue that present." "Drainage of the foot and instilling antibiotics into the affected area are the usual therapy."

Correct Answer: "IV antibiotics are usually required for several weeks." Rationale: The standard treatment for acute osteomyelitis consists of several weeks of IV antibiotic therapy. However, as many as 3 to 6 months may be required. Bone is denser and less vascular than other tissues, and it takes time for the antibiotic therapy to eradicate all microorganisms. Surgery may be used for chronic osteomyelitis, to include debridement of the devitalized, infected tissue and irrigation of the affected bone with antibiotics.

A nurse performs discharge teaching for a patient after a left hip arthroplasty using the posterior approach. Which statement indicates teaching is successful?

Correct Answer: "Leg-raising exercises are necessary for several months." Rationale: Exercises designed to restore strength and muscle tone will be done for months after surgery. The exercises include leg raises in supine and prone positions. Driving a car is not allowed for 4 to 6 weeks. In the posterior approach hip arthroplasties, extremes of internal rotation and 90-degree flexion of the hip must be avoided for 4 to 6 weeks postoperatively. The knees must be kept apart. The patient should never cross the legs or twist to reach behind. To prevent thromboembolism, enoxaparin is administered subcutaneously and can be given at home. Enoxaparin does not require monitoring of the patient's coagulation status.

The nurse receives report from the licensed practical nurse (LPN/VN) about care provided to patients on the orthopedic surgical unit. It is most important for the nurse to follow up on which statement? "The patient who had a spinal fusion 12 hours ago has hypoactive bowel sounds and is not passing flatus." "The patient who had cervical spine surgery 2 days ago wants to wear her soft cervical collar when out of bed." "The patient who had spinal surgery 3 hours ago is reporting a headache and has clear drainage on the dressing." "The patient who had a laminectomy 24 hours ago is using patient-controlled analgesia with morphine for pain management."

Correct Answer: "The patient who had spinal surgery 3 hours ago is reporting a headache and has clear drainage on the dressing." Rationale: After spinal surgery, there is potential for cerebrospinal fluid (CSF) leakage. Severe headache or leakage of CSF (clear or slightly yellow) on the dressing should be reported immediately. The drainage is CSF if a dipstick test is positive for glucose. Patients after spinal surgery may experience interference with bowel function for several days. Postoperatively most patients require opioids such as morphine IV for 24 to 48 hours. Patient-controlled analgesia is the preferred method for pain management during this time. After cervical spine surgery, patients often wear a soft or hard cervical collar to immobilize the neck.

The nurse is caring for patients in a primary care clinic. Which patient is most at risk to develop osteomyelitis caused by Staphylococcus aureus? A 22-yr-old female patient with gonorrhea who is an IV drug user A 48-yr-old male patient with muscular dystrophy and acute bronchitis A 32-yr-old male patient with type 1 diabetes and stage 4 pressure injury A 68-yr-old female patient with hypertension who had a knee arthroplasty 3 years ago

Correct Answer: A 32-yr-old male patient with type 1 diabetes and stage 4 pressure injury Rationale: Osteomyelitis caused by S. aureus is usually associated with a pressure ulcer or vascular insufficiency related to diabetes. Osteomyelitis caused by Staphylococcus epidermidis is usually associated with indwelling prosthetic devices from joint arthroplasty. Osteomyelitis caused by Neisseria gonorrhoeae is usually associated with gonorrhea. Osteomyelitis caused by Pseudomonas is usually associated with IV drug use. Muscular dystrophy is not associated with osteomyelitis.

A patient presents to the clinical after tripping on a curb and spraining the right ankle. Which initial care measures are appropriate? (Select all that apply.) Apply ice directly to the skin. Apply heat to the ankle every 2 hours. Administer antiinflammatory medication. Compress ankle using an elastic bandage. Rest and elevate the ankle above the heart. Perform passive and active range of motion.

Correct Answer: Administer antiinflammatory medication. Compress ankle using an elastic bandage. Rest and elevate the ankle above the heart. Rationale: Appropriate care for a sprain is represented with the acronym RICE (rest, ice, compression, and elevation). Antiinflammatory medication should be used to decrease swelling if not contraindicated for the patient. After the injury, the ankle should be immobilized and rested. Prolonged immobilization is not required unless there is significant injury. Ice is indicated but will cause tissue damage if applied directly to the skin. Apply ice to sprains as soon as possible and leave in place for 20 to 30 minutes at a time. Moist heat may be applied 24 to 48 hours after the injury.

The nurse is admitting a patient with a history of a herniated lumbar disc and low back pain. Which action would likely aggravate the pain?Bending or liftingApplication of warm moist heatSleeping in a side-lying positionSitting in a fully extended recliner

Correct Answer: Bending or lifting Rationale: Back pain related to a herniated lumbar disc is aggravated by events and activities that increase stress and strain on the spine, such as bending or lifting, coughing, sneezing, and lifting the leg with the knee straight (straight leg-raising test). Moist heat, sleeping position, and ability to sit in a fully extended recliner do not aggravate the pain of a herniated lumbar disc.

A patient who has low back pain from a herniated lumbar disc is having muscle spasms. Which nursing intervention would be most appropriate? Provide gentle ROM to the lower extremities. Elevate the head of the bed 20 degrees and flex the knees. Place a small pillow under the patient's upper back to gently flex the lumbar spine. Place the bed in reverse Trendelenburg with the patient's feet against the footboard.

Correct Answer: Elevate the head of the bed 20 degrees and flex the knees. Rationale: To reduce pain, the nurse should elevate the head of the bed 20 degrees and have the patient flex the knees to avoid extension of the spine. The slight flexion provided by this position often is comfortable for a patient with a herniated lumbar disc. ROM to the lower extremities will be limited to prevent extremes of spinal movement. Reverse Trendelenburg and a pillow under the patient's upper back will more likely increase pain.

The nurse determines that an older adult patient recovering from left total knee arthroplasty has impaired physical mobility from decreased muscle strength. What nursing intervention is appropriate? Promote vitamin C and calcium intake in the diet. Provide passive range of motion to all the joints every 4 hours. Keep the left leg in extension and abduction to prevent contractures. Encourage isometric quadriceps-setting exercises at least 4 times a day.

Correct Answer: Encourage isometric quadriceps-setting exercises at least 4 times a day. Rationale: Emphasis is placed on postoperative exercise of the affected leg, with isometric quadriceps setting beginning on the first day after surgery. Vitamin C and calcium do not improve muscle strength, but they will facilitate healing. The patient should be able to perform active range of motion to all joints. Keeping the leg in one position (extension and abduction) may contribute to contractures.

The patient brought to the emergency department after a car accident is diagnosed with a femur fracture. What nursing intervention should the nurse implement at this time to decrease risk of a fat embolus?

Correct Answer: Immobilize the fracture preoperatively. Rationale: The nurse immobilizes the long bone to reduce movement of the fractured bone ends and decrease the risk of a fat embolus development before surgical reduction. Enoxaparin is used to prevent blood clots, not fat emboli. Range of motion and compression boots will not prevent a fat embolus in this patient.

The nurse is caring for a patient with osteoarthritis scheduled for total left knee arthroplasty. Preoperatively, the nurse assesses for which contraindication to surgery? Pain Left knee stiffness Left knee infection Left knee instability

Correct Answer: Left knee infection Rationale: The patient must be free of infection before total knee arthroplasty. An infection in the joint could lead to even greater pain and joint instability, requiring more extensive surgery. The nurse must assess the patient for signs of infection, such as redness, swelling, fever, and elevated white blood cell count. Pain, knee stiffness, or instability are typical of osteoarthritis.

The nurse is caring for a patient admitted to the nursing unit with osteomyelitis of the tibia. When completing a focused assessment, which symptom should the nurse expect? Nausea and vomiting Localized pain and warmth Paresthesia in the affected extremity Generalized bone pain throughout the leg

Correct Answer: Localized pain and warmth Rationale: Osteomyelitis is an infection of bone and bone marrow that can occur with trauma, surgery, or spread from another part of the body. Because it is an infection, the patient will exhibit typical signs of inflammation and infection, including localized pain and warmth. Nausea and vomiting and paresthesia of the extremity are not expected to occur. Pain occurs, but it is localized rather than generalized throughout the leg.

A patient with a fracture of the proximal left tibia in a long leg cast reports of severe pain and a prickling sensation in the left foot. The toes on the left foot are pale and cool. Which nursing action is a priority? Notify the health care provider immediately. Elevate the left leg above the level of the heart. Administer prescribed morphine sulfate intravenously. Apply ice packs to the left proximal tibia over the cast.

Correct Answer: Notify the health care provider immediately. Rationale: Notify the health care provider immediately of this change in patient's condition, which suggests development of compartment syndrome. Pain unrelieved by drugs and out of proportion to the level of injury is one of the first indications of impending compartment syndrome. Changes in sensation (tingling) also suggest compartment syndrome. Because elevation of the extremity may lower venous pressure and slow arterial perfusion, the extremity should not be elevated above heart level. Similarly, the application of cold compresses may result in vasoconstriction and exacerbate compartment syndrome. Administration of morphine may be warranted, but it is not the first priority.

A 67-yr-old patient hospitalized with osteomyelitis has an order for bed rest with bathroom privileges and elevation of the affected foot on 2 pillows. The nurse would place the highest priority on which intervention? Ambulate the patient to the bathroom every 2 hours. Ask the patient about preferred activities to relieve boredom. Allow the patient to dangle legs at the bedside every 2 to 4 hours. Perform frequent position changes and range-of-motion exercises.

Correct Answer: Perform frequent position changes and range-of-motion exercises. Rationale: The patient is at risk for atelectasis of the lungs and contractures because of prescribed bed rest. For this reason, the nurse should place the priority on changing the patient's position frequently to promote lung expansion and performing range-of-motion exercises to prevent contractures. Assisting the patient to the bathroom will keep the patient safe as the patient is in pain, but it may not be needed every 2 hours. Providing activities to relieve boredom will assist the patient to cope with the bed rest. Dangling the legs every 2 to 4 hours may be too painful.

Which nursing intervention is most appropriate when turning a patient after spinal surgery? Having the patient turn to the side by grasping the side rails to help turn Placing a pillow between the patient's legs and turning the body as a unit Elevating the head of bed 30 degrees and having the patient extend the legs while turning Turning the patient's head and shoulders and then the hips, keeping the patient's body centered in the bed

Correct Answer: Placing a pillow between the patient's legs and turning the body as a unit Rationale: Placing a pillow between the legs and turning the patient as a unit (logrolling) helps to keep the spine in good alignment and reduces pain and discomfort after spinal surgery. The other interventions will not maintain proper spine alignment and may cause spinal damage.

When the patient is diagnosed with muscular dystrophy, what information should the nurse include in the teaching plan?

Correct Answer: Remain active to prevent skin breakdown and respiratory complications. Rationale: With muscular dystrophy, the patient must remain active for as long as possible. Prolonged bed rest should be avoided because immobility leads to further muscle wasting. An orthotic jacket may be used to provide stability and prevent further deformity. Continuous positive airway pressure (CPAP) may be used as respiratory function decreases before mechanical ventilation is needed to sustain respiratory function.

The nurse completes an admission history for a 73-yr-old man with osteoarthritis scheduled for total knee arthroplasty. Which response is expected when asking the patient the reason for admission? Recent knee trauma Debilitating joint pain Repeated knee infections Onset of frozen knee joint

Debilitating joint pain Rationale: The most common reason for knee arthroplasty is debilitating joint pain despite exercise, weight management, and drug therapy. Recent knee trauma, repeated knee infections, and onset of frozen knee joint are not primary indicators for a knee arthroplasty.

The nurse is completing discharge teaching with a patient who is recovering from a right total hip arthroplasty by posterior approach. Which patient action indicates further instruction is needed? Uses an elevated toilet seat. Sits with feet flat on the floor. Maintains hip in adduction and internal rotation. Verifies need to notify future caregivers about the prosthesis.

Maintains hip in adduction and internal rotation. Rationale: The patient should not force hip into adduction or internal rotation because these movements could dislocate the hip prosthesis. Sitting with feet flat on the floor (avoiding crossing the legs), using an elevated toilet seat, and notifying future caregivers about the prosthesis indicate understanding of discharge teaching.

The nurse is caring for a patient who had a left total knee arthroplasty to relieve the pain of severe osteoarthritis. What care would be expected postoperatively? Progressive leg exercises to obtain 90-degree flexion Early ambulation with full weight bearing on the left leg Bed rest for 3 days with the left leg immobilized in extension Immobilization of the left knee in 30-degree flexion to prevent dislocation

Progressive leg exercises to obtain 90-degree flexion Rationale: The patient is encouraged to engage in progressive leg exercises until 90-degree flexion is possible; continuous passive motion also may be used based on surgeon preference. Early ambulation is implemented, sometimes the day of surgery, but orders are likely to indicate weight bearing as tolerated rather than full weight bearing. Immobilization and bed rest are not indicated. The patient's knee is unlikely to dislocate.

What should the nurse teach the patient recovering from an episode of acute low back pain a. perform daily exercise as a lifelong routine b. Sit in a chair with the hips higher than the knees c. Avoid occupations in which the use of the body is required d. Sleep on the abdomen or on the back with the legs extended

a Proper daily stretching and strengthening exercises are an important part of the prevention of back injury, with the goal of maintaining mobility and strength in the back. Patients should sit with the knees level with the hips and should sleep in a side-lying position, with knees and hips bent, or on the back, with a device to flex the hips and knees. Good body mechanics with proper transfer and turning techniques are necessary in all jobs and activities.

Which individuals would be at high risk for low back pain... (select all that apply) a. A 63-year-old man who is a long-distance truck driver b. A 36-year-old construction worker who is 6 ft 2 in and weighs 260 lb c. A 44-year-old female chef with prior compression fracture of the spine d. A 28-year-old female yoga instructor who is 5 ft 6 in and weighs 130 lb.

a. A 63-year-old man who is a long-distance truck driver b. A 36-year-old construction worker who is 6ft 2 in and weighs 260 lb c. A 44-year-old female chef with prior compression fracture of the spine d. A 28-year-old female yoga instructor who is 5ft 6 in and weighs 130 lb. Rationale: .Risk factors associated with low back pain include lack of muscle tone, excess body weight, stress, poor posture, cigarette smoking, pregnancy, prior spinal compression fractures, spinal problems since birth, and a family history of back pain. Jobs that require repetitive heavy lifting, vibration (such as a jackhammer operator), and prolonged sitting are also associated with low back pain. Low back pain is most often caused by a musculoskeletal problem. Causes of low back pain of musculoskeletal origin include (1) acute lumbosacral strain, (2) instability of the lumbosacral spine, (3) osteoarthritis of the lumbosacral vertebrae, (4) degenerative disc disease, and (5) herniated disc. Health care personnel are at high risk for low back pain. Lifting and moving patients, excessive time being stooped over or leaning forward, and frequent twisting can cause low back pain.

A patient with a pelvic fracture should be monitored for... a. changes in urine output b. petechiae on the abdomen c. a palpable lump in the buttock d. sudden increase in blood pressure

a. changes in urine output Rationale: Pelvic fractures may cause serious intraabdominal injury, such as hemorrhage, and laceration of the urethra, bladder, or colon. Patients may survive the initial pelvic injury, only to die of sepsis, FES, or VTE. Because a pelvic fracture can damage other organs, the nurse should assess bowel and urinary elimination and distal neurovascular status.

A patient with a humeral fracture is returning for a 4-week checkup. The nurse explains that initial evidence of healing on x-ray is indicated by... a. formation of callus b. complete bony union c. hematoma at the fracture site d. presence of granulation tissue

a. formation of callus Rationale: Bone goes through a remarkable reparative process of self-healing (i.e. union) that occurs in stages. The third stage is callus formation. As minerals (i.e. calcium, phosphorus, and magnesium) and new bone matrix are deposited in the osteoid, an unorganized network of bone is formed that is woven about the fracture parts. Callus is composed primarily of cartilage, osteoblasts, calcium, and phosphorus. It usually appears by the end of the second week after injury. Evidence of callus formation can be verified on x-rays.

A patient with chronic osteomyelitis has been hospitalized for a surgical debridement procedure. What does the nurse explain to the patient as the rationale for the surgical treatment. a. Removal of the infection prevents the need for bone and skin grafting b. Formation of scar tissue has led to protected area of bacterial growth c. The process of depositing new bone blacks the vascular supply to the bone d. Antibiotics are not effective against microorganisms that cause chronic osteomyelitis

b Chronic infection of the bone leads to formation of scar tissue from granulation tissue. This avascular scar tissue provides an ideal site for continued microorganism growth and is impenetrable to antibiotics. Surgical debridement is often necessary to remove the poorly vascularized tissue and dead bone, and to instill antibiotics directly to the area. Bone and skin grafting may be needed after surgical removal of infection if destruction is extensive. Involucrum is new bone laid down at the infection site, which seals off areas of dead bone (sequestra) that may hold microorganisms that spread to other sites. Antibiotics can be effective during acute osteomyelitis. Prevention of chronic osteomyelitis requires early antibiotic treatment.

A patient with osteomyelitis has a nursing diagnosis of risk for injury. What is an appropriate nursing intervention for this patient a. Use careful and appropriate disposal of soiled dressings b. Gently handle the involved extremity during movement c. Measure the circumference of the affected extremity daily d. Range-of-motion exercise every 4 hours to the involved extremity.

b The patient with osteomyelitis is at risk for pathologic fractures at the infection site because of weakened, devitalized bone so careful handling of the extremity is needed. Careful handling of dressings is needed to prevent the spread of infection to othersbut is not related to preventing injury to this patient. Splints may be used to immobilize the limb. Edema is not a common finding in osteomyelitis. Range-ofmotion (ROM) exercises will be limited because of the possibility of spreading infection.

A laminectomy and spinal fusion are performed on a patient with a herniated lumbar intervertebral disc. During the postoperative period, which finding is of most concern to the nurse a. Paralytic ileus b. Urinary incontinence c. Greater pain at the graft site than at the lumber incision site d. Leg and arm movement and sensation unchanged from preoperative status

b . Urinary incontinence after spinal surgery may indicate nerve damage and should be reported to the HCP. Paralytic ileus is not unexpected after this type of surgery. Pain at the graft site, usually the iliac crest or the fibula, is more often severe than pain at the fused area. Although movement and sensation of the arms and legs should not be more impaired than before surgery, deficits are not often relieved immediately after surgery.

Priority Decision: During a follow-up visit to a patient with acute osteomyelitis treated with IV antibiotics, the home health nurse is told by the patient's wife that she can hardly get the a patient to eat because his mouth is so sore. In assessing the patient's mouth, what is the most likely finding that the nurse should expect to find a. A dry, cracked tongue with central furrow b. white, curdlike membranous lesions of the mucosa c. Ulcers of the mouth and lips surrounded by a reddened base d. Single or clustered vesicles on the tongue and buccal mucosa

b One of the most common adverse effects of prolonged and high-dose antibiotic therapy is overgrowth of Candida albicans in the oral cavity and genitourinary tract. These infections are marked by whitish-yellow, curdlike lesions of the mucosa. A dry, cracked, furrowed tongue is characteristic of severe dehydration and vesicles are characteristic of herpes simplex infections. Mouth and lip ulcers are characteristic of aphthous stomatitis (canker sores).

Patient-Centered Care: A 24 year old patient with a 12 year history of Becker muscular dystrophy is hospitalized with heart failure. What is an appropraite nursing intervention for this patient. a. Feed and bathe the patient b. Reposition frequently to avoid skin and respiratory complications c. Provide hand weighs for the patient to exercise the upper extremities d. Use orthopedic braces to promote ambulations and prevent muscle wasting

b Promoting muscle activity is important in any patient with muscular dystrophy. However, when the disease has progressed to cardiomyopathy or respiratory failure, activity must be balanced with oxygen supply. At this stage of the disease, care should be taken to prevent skin or respiratory complications. The patient should be encouraged to perform as much self-care and exercise as energy allows, but this will be limited.

A patient with osteomyelitis undergoes surgical debridement with implantation of antibiotic beads. When the patient asks why the beads are used, the nurse answers... (select all that apply) a. "Oral or IV antibiotics are not effective in most cases of bone infection." b. "The beads are an adjunct to debridement and antibiotics for deep infections." c. "The beads are used to deliver antibiotics directly to the site of the infection." d. "This is the safest method to deliver long-term antibiotic therapy for bone infection." e. "Ischemia and bone death related to osteomyelitis are impenetrable to IV antibiotics."

b. "The beads are an adjunct to debridement and antibiotics for deep infections." c. "The beads are used to deliver antibiotics directly to the site of the infection." Rationale: Treatment of chronic osteomyelitis includes surgical removal of the poorly perfused tissue and dead bone in addition to the extended use of IV and oral antibiotics. Antibiotic acrylic bead chains may be placed during surgery to help fight the infection.

In caring for a patient after a spinal fusion, the nurse would immediately report which of the following to the surgeon? a. The patient experiences a single episode of emesis b. The patient is unable to move lower extremities c. The patient is nauseated and has not voided in 4 hours d. The patient complains of pain at the bone graft donor site

b. The patient is unable to move lower extremities. ationale: After spinal fusion surgery, the nurse should frequently monitor peripheral neurovascular condition. Movement of the arms and legs and assessment of sensation should be no worse in comparison with the preoperative status. These assessments are repeated at least every 2 to 4 hours during the first 48 hours after surgery, and findings are compared with the preoperative assessment. Paresthesia, such as numbness and tingling, may not be relieved immediately after surgery. The nurse should immediately report any new muscle weakness or paresthesia to the surgeon and document this in the patient's medical record.

A patient with osteoarthritis is scheduled for a total hip arthroplasty. The nurse explains the purpose of this procedure is to... (select all that apply) a. fuse the joint b. replace the joint c. prevent further damage d. improve or maintain ROM e. decrease the amount of destruction in the joint

b. replace the joint d. improve or maintain ROM Rationale: Arthroplasty is the reconstruction or replacement of a joint. This surgical procedure is performed to relieve pain, improve or maintain range of motion, and correct deformity. Total hip arthroplasty (THA) provides significant pain relief and improved function for a patient with osteoarthritis (OA).

A patient who experienced an open fracture of humerus 2 weeks ago is having increased pain at the fracture site. To identify a possible causative agent of osteomyelitis at the site, what should the nurse expect testing to include a. x-rays b. CT scan c. Bone Biopsy d. WBC count and erythrocyte sedimentation rate (ESR)

c Because large doses of appropriate antibiotics are needed in the treatment of acute osteomyelitis, it is important to identify the causative microorganism. The definitive way to determine the causative agent is by bone biopsy or biopsy of the soft tissue surrounding the site. The other tests may help establish the diagnosis but do not identify the causative agent

The nurse suspects a neurovascular problem based on assessment of... a. exaggerated strength with movement b. increased redness and heat below the injury c. decreased sensation distal to the fracture site d. purulent drainage at the site of an open fracture

c. decreased sensation distal to the fracture site Rationale: Musculoskeletal injuries have the potential for causing changes in the neurovascular condition of an injured extremity. Application of a cast or constrictive dressing, poor positioning, and physiologic responses to the injury can cause nerve or vascular damage, usually distal to the injury. The neurovascular assessment consists of peripheral vascular evaluation (i.e. color, temperature, capillary refill, peripheral pulses, and edema) and peripheral neurologic evaluation (i.e. sensation and motor function).

Following 7 days of IV antibiotic therapy, a patient with acute osteomyelitis of the tibia is prepared for discharge from the hospital. The nurse determines that additional instruction is needed when the patient makes which statement a. I will need to continue antibiotic therapy for 4 to 6 weeks b. I should notify the HCP if the pain in my leg becomes worse c. I shouldn't bear weight on my affected leg until healing is complete d. I do not need to do anything special while taking the antibiotic therapy

d Activities, such as exercises that increase circulation and serve as stimuli for the spread of infection, should be avoided by patients with acute osteomyelitis. Oral or IV antibiotic therapy is continued at home for 4 to 6 weeks. The HCP should be notified if increased pain occurs. Weight bearing is contraindicated to prevent pathologic fractures. Monitoring for side effects and complications of antibiotic therapy must be done.

What does radicular pain that radiates down the buttock and below the knee, along the distribution of the sciatic nerve, generally indicate a. Cervical disc herniation b. acute lumboscaral strain c. Degenerative disc disease d. Herniated lumbar disc disease

d Lumbar disc herniation is generally indicated by radicular pain radiating down the buttock, below the knee, and along the distribution of the sciatic nerve. Cervical disc disease has pain radiating into the arms and hands. Acute lumbosacral strain causes acute low back pain. Degenerative disc disease is a structural degeneration of discs that often occurs with aging and results in intervertebral discs losing their elasticity,flexibility, and shock-absorbing capabilities

A patient with a comminuted fracture of the tibia is to have an open reduction with internal fixation (ORIF) of the fracture. The nurse explains that ORIF is indicated when... a. the patient is unable to tolerate prolonged immobilization b. the patient cannot tolerate the surgery for a closed reduction c. a temporary cast would be too unstable to provide normal mobility d. adequate alignment cannot be obtained by other nonsurgical methods

d. adequate alignment cannot be obtained by other nonsurgical methods Rationale: A comminuted fracture has more than two bone fragments. Open reduction with internal fixation (ORIF) is indicated for a comminuted fracture and is used to realign and maintain bony fragments. Other nonsurgical methods can result in a failure to obtain satisfactory reduction. Internal fixation reduces the hospital stay and complications associated with prolonged bed rest.

A patient is scheduled for total ankle replacement. The nurse should tell the patient that after surgery he should avoid... a. lifting heavy objects b. sleeping on the back c. abduction exercises of the affected ankle d. bearing weight on the affected leg for 6 weeks

d. bearing weight on the affected leg for 6 weeks Rationale: After total ankle arthroplasty (TAA), the patient may not bear weight for 6 weeks and must elevate the extremity to reduce edema. The patient must follow strategies to prevent postoperative infection and maintain immobilization as directed by the surgeon.

A patient with a stable, closed humeral fracture has a temporary splint with bulky padding applied with an elastic bandage. The nurse notifies the surgeon of possible early compartment syndrome when the patient experiences... a. increasing edema of the limb b. muscle spasms of the lower arm c. bounding pulse at the fracture site d. pain when passively extending the fingers

d. pain when passively extending the fingers Rationale: One or more of the following are characteristic of early compartment syndrome: (1) paresthesia (numbness and tingling sensation); (2) pain distal to the injury that is not relieved by opioid analgesics and is increased on passive stretch of muscle; (3) increased pressure in the compartment; and (4) pallor, coolness, and loss of normal color of the extremity. Paralysis (or loss of function) and pulselessness (or diminished or absent peripheral pulses) are late sign of compartment syndrome. The examination also includes assessment of peripheral edema, especially pitting edema, which may occur with severe injury.

A patient with disc herniation is experiencing acute pain and muscle spasms. The nurse's responsibility is to... a. encourage total bed rest for several days b. teach principles of back strengthening exercises c. stress the importance of straight-leg raises to decrease pain d. promote use of cold and hot compresses and pain medication

d. promote use of cold and hot compresses and pain medication Rationale: If acute pain and muscle spasms are not severe, the patient may be treated as an outpatient with NSAIDs and muscle relaxants (e.g., cyclobenzaprine [Flexeril]). Massage and back manipulation, acupuncture, and application of cold and hot compresses may help some patients. Severe pain may require a brief course of opioid analgesics. A brief period (1 to 2 days) of rest at home may be needed for some people; most patients do better if they continue their regular activities. Prolonged bed rest should be avoided. All patients should refrain from activities that aggravate the pain, including lifting, bending, twisting, and prolonged sitting.

The nurse suspects an ankle sprain when a patient at the urgent care center describes... a. being hit by another soccer player during a game b. having ankle pain after sprinting around the track c. dropping a 10-lb weight on his lower leg at the health club d. twisting his ankle while running bases during a baseball game

d. twisting his ankle while running bases during a baseball game Rationale: A sprain is an injury to the ligaments surrounding a joint and is usually caused by a wrenching or twisting motion. Most sprains occur in the ankle and knee joints.


Ensembles d'études connexes

Combo with "Peter the Great" and 1 other

View Set

Personality, Ch. 10 (from Launchpad)

View Set

Describe core Azure architectural components

View Set